Hippo Ed: Internal Medicine

Ace your homework & exams now with Quizwiz!

Sexual history **classic triad of reactive arthritis (arthritis, urethritis, and uveitis)

A 19-year-old man comes to the primary care clinic because he has had worsening crampy abdominal pain over the past two days. He also has had subjective fevers, chills, achy swollen joints, and eye pain. The patient has noticed a tingling sensation deep within his perineum. He has not had diarrhea or constipation. On physical examination, a red ring is noted around the iris of both eyes. Severe edema of the fingers of the right hand is noted. Based on these findings, it is most appropriate to ask about which of the following aspects of this patient's history? A. Dietary history B. Family history C. Sexual history D. Social history E. Travel history

CT scan of the head **meningitis to evaluate for mass effect or hemorrhage

A 19-year-old man who is in the military and lives in the barracks is brought to the emergency department by ambulance because he has had worsening fever, chills, and changes in mentation over the past 12 hours. He initially had a headache, dizziness, as well as nausea and vomiting last night, but this progressed throughout the night to generalized myalgia and neck stiffness. Medical history includes no chronic disease conditions, and he takes no medications. The patient drinks alcoholic beverages on occasion but does not smoke cigarettes or use illicit drugs. Temperature is 38.9°C (102.0°F), pulse rate is 125/min, and blood pressure is 108/78 mmHg. Glasgow Coma Scale score is 11/15. The patient appears toxic and is not able to answer questions due to confusion. Cardiac, abdominal, and pulmonary examinations show no abnormalities. On physical examination, flexion of the head while supine causes the hips and knees to lift. Straightening the legs while supine with the hips flexed elicits pain. Funduscopic examination shows an elevated optic disc with dilated veins. White blood cell count is 21/mm³, sodium level is 140 mEq/L, and creatinine level is 1.2 mg/dL. Which of the following is the most appropriate next step in diagnosis? A. CT scan of the head B. Electroencephalography C. Lumbar puncture D. MRI of the brain E. Slit-lamp examination

Administer hydromorphone **sickle cell disease vaso-occlusive crisis that needs to be treated with pain control and fluids

A 19-year-old patient of Mediterranean descent comes to the emergency department because he has had severe hip and leg pain during the past six hours. He says that the pain started in the middle of the night and has been increasing in intensity; he cannot handle the pain anymore and rates it as an 11 on a 10-point scale. The pain is deep and squeezing in both legs and hips. The patient says he may have pulled something because he has been training for the police academy and did some running in the mountains the day before. He says he had a similar episode in the past, and the only treatment that helped was a drug that starts with a "D." Medical history includes no chronic disease conditions, and he takes no medications. Temperature is 37.0°C (98.6°F), pulse rate is 108/min, and blood pressure is 104/62 mmHg. Oxygen saturation is 96%. The patient appears mildly uncomfortable. Physical examination shows full range of motion of the extremities and no tenderness to palpation. Laboratory studies show white blood cell count of 9000/mm³, hemoglobin level of 12.9 g/dL, and hematocrit of 38%. Peripheral blood smear shows a few abnormally shaped cells as well as polychromatophilic red blood cells, target cells, and Howell-Jolly bodies. A 1L fluid bolus of 9% normal saline is administered. Which of the following is the most appropriate next step? A. Administer hydromorphone B. Administer nitric oxide C. Administer piperacillin/tazobactam D. No treatment is necessary E. Recommend rest, ice, compression, and elevation of the legs

Pulmonary function testing **exercise-induced asthma

A 21-year-old man comes to the clinic because he has had shortness of breath and cough for the past 3 months. He says he is trying to get into the police academy and has been running in order to pass the physical requirement. He has to stop his runs early because he is getting extremely short of breath. He has not tried anything to improve these symptoms. The patient does not remember having this issue before but has generally avoided running in the past. He has not had fever, chills, chest pain, or peripheral edema. Medical history does not include any recent illnesses. The patient has never smoked cigarettes, drinks alcoholic beverages socially, and does not use illicit drugs. Temperature is 36.5°C (97.7°F), pulse rate is 62/min, respirations are 12/min, and blood pressure is 110/80 mmHg. Oxygen saturation is 99%. On physical examination, the lung are clear to auscultation. Cardiac examination shows a regular rate without murmurs, gallops, or rubs. Abdominal examination shows no abnormalities. No edema is noted. Which of the following studies is most likely to determine the diagnosis? A. Chest x-ray study B. Complete blood cell count with differential C. CT scan of the chest D. Electrocardiography E. Pulmonary function testing

Chlamydia trachoma's **epididymitis from STI (chlamydia is the most common cause -- treat with ceftriaxone and doxycycline)

A 22-year-old man comes to the internal medicine clinic because he has had gradual pain and swelling in the right testicle for the past four days. He says he has a new sexual partner. Medical history includes several sexually transmitted infections, but he is otherwise healthy. Temperature is 37.1°C (98.8°F), pulse rate is 80/min, respirations are 16/min, and blood pressure is 128/87 mmHg. Oxygen saturation is 98% on room air. On physical examination, the right scrotum and testicle are erythematous, tender to palpation, and indurated. Tenderness is relieved with elevation of the right scrotum, and cremasteric reflex is within normal limits. Ultrasonography shows an enlarged epididymis with increased testicular blood flow. Urinalysis is positive for leukocyte esterase and white blood cells. Which of the following is the most likely causative agent? A. Chlamydia trachomatis B. Coxsackievirus C. Escherichia coli D. Mumps E. Ureaplasma urealyticum

Cervical lymphadenopathy ** Hodgkin lymphoma

A 22-year-old woman comes to the clinic for follow-up to review the results of recent laboratory studies. She initially presented to the clinic because she had been having fatigue and weight loss for six weeks. Peripheral blood smear indicates that she has Reed-Sternberg cells. Which of the following findings on physical examination is most likely? A. Cervical lymphadenopathy B. Friable cervix C. Hematochezia D. Hemopytysis E. Thyroid mass

Diabetic ketoacidosis

A 24-year-old woman is brought to the emergency department by her roommate because she had acute onset of abdominal pain as well as confusion, nausea and vomiting, and weakness 12 hours ago. The roommate does not believe that the patient has any medical conditions or takes any medications. Body mass index is 22 kg/m². Pulse rate is 110/min, and respirations are 28/min. The patient appears flushed and is oriented only to person and place. Physical examination shows decreased skin turgor. Laboratory studies show the following: plasma glucose 304 mg/dL; arterial pH 7.2; serum bicarbonate 14 mEq/L; thyroid-stimulating hormone 2.5 µU/L; and free thyroxine 1.4 µg/dL. Urinalysis is positive for ketones. Intravenous fluids are initiated, and the patient is admitted to the internal medicine ward. Which of the following is the most likely diagnosis? A. Acute adrenal crisis B. Diabetic ketoacidosis C. Hyperosmolar hyperglycemia D. Myxedema coma E. Thyroid storm

Reactive arthritis **Reiter syndrome occurs after an extra-articular infection from STI

A 26-year-old man comes to the internal medicine clinic because he has had redness of the eyes with associated discharge, pain with urination, and pain in his right knee and right ankle for the past five days. He says he is generally healthy and has no chronic disease conditions; however, he had Chlamydia trachomatis about five weeks ago. Temperature is 37.1°C (98.8°F), pulse rate is 80/min, respirations are 16/min, and blood pressure is 128/87 mmHg. Oxygen saturation is 98% on room air. Physical examination shows conjunctivitis, edema and tenderness to palpation of the right knee and ankle, and urethral discharge. Laboratory studies show elevated C-reactive protein level and erythrocyte sedimentation rate. Which of the following is the most likely diagnosis? A. Ankylosing spondylitis B. Pseudogout C. Reactive arthritis D. Rheumatoid arthritis E. Septic arthritis

Splinting of the wrist for 2-6 weeks **mild-moderate case of carpal tunnel syndrome

A 26-year-old woman comes to the internal medicine clinic because she has had tingling and burning in her left wrist and hand for the past three weeks. She works as a stenographer and spends most of her day typing. Temperature is 37.1°C (98.8°F), pulse rate is 80/min, respirations are 16/min, and blood pressure is 128/87 mmHg. Oxygen saturation is 98% on room air. On physical examination, Phalen test is positive. No atrophy of the thenar eminence is noted. Which of the following is the most appropriate initial step in management? A. Injection of corticosteroids B. Prescription for alendronate C. Prescription for gabapentin D. Referral for surgical consultation E. Splinting of the wrist for two to six weeks

Recent viral illness **myocarditis is inflammation of the heart muscle and is commonly caused by a viral infection

A 28-year-old patient who identifies as non-binary comes to the internal medicine clinic because of shortness of breath and palpitations that have progressively worsened over the past month. The shortness of breath is most noticeable with light jogging. The patient is otherwise healthy, and medical history includes no chronic disease conditions. Physical examination shows no abnormalities. Electrocardiography shows nonspecific changes. Chest x-ray study shows cardiomegaly. Myocarditis is suspected. Which of the following additional findings on history taking is most likely to support this diagnosis? A. Family history of myocarditis B. Recent medication changes C. Recent viral illness D. Rheumatic fever as a child E. Underlying autoimmune disease

Intravenous antibiotics **infective bacterial endocarditis from IV drug use affecting the tricuspid valve

A 28-year-old woman is admitted to the hospital after she was evaluated in the emergency department because of persistent fevers for the past six days. Medical history does not include any chronic disease conditions, but she has a history of intravenous drug use. Temperature is 38.0°C (100.4°F); all other vital signs are within normal limits. On physical examination, a new systolic murmur is heard best at the lower left sternal border. Echocardiography shows tricuspid vegetations. Which of the following is the most appropriate management? A. Anticoagulation B. Intravenous antibiotics C. Septal myectomy D. Supportive care E. Valve replacement

Supportive care while monitoring salicylate and acetaminophen levels

A 28-year-old woman with major depressive disorder and a history of suicidal ideation is admitted to the hospital after being found in the bathroom with an empty bottle of pills. The patient is tearful and admits that she wanted to end things and took a handful of pills she found in the bathroom cabinet about four hours ago. She says she has vomited several times and has generalized abdominal pain. Pulse rate is 90/min, respirations are 26/min, and blood pressure is 110/80 mmHg; the patient is afebrile. On physical examination, mild tenderness to palpation is noted in all quadrants of the abdomen. Extensive laboratory studies are obtained including salicylate, acetaminophen, and electrolyte levels, complete blood cell count, lactate level, arterial blood gas analysis, and liver and renal function tests. All results are within normal limits except for arterial blood gas analysis showing mild respiratory alkalosis and an elevated salicylate level of 50 mg/dL. Electrocardiography shows no abnormalities. Which of the following is the most appropriate next step in management? A. Administration of benzodiazepines B. Hemodialysis C. Induce vomiting D. Intubation E. Supportive care while monitoring salicylate and acetaminophen levels

G6PD deficiency **hemolytic anemia triggered by fava beans

A 29-year-old man comes to the clinic because he has had fatigue since he returned from a trip to Angola one week ago. He says he also has noticed that his urine has been darker, and his belly seems larger on the left side. The patient has not had fevers, chills, or changes in weight. He recently started eating a vegetarian diet and has been eating fava beans frequently. While in Angola, he took antimalarial prophylaxis. Medical history does not include any chronic disease conditions. The patient does not drink alcoholic beverages, smoke cigarettes, or use illicit drugs. Temperature is 37.0°C (98.6°F), pulse rate is 80/min, and blood pressure is 116/80 mmHg. Oxygen saturation is 96%. On physical examination, the skin appears slightly jaundiced. Abdominal examination shows splenomegaly. Peripheral blood smear shows schistocytes, Heinz bodies, and bite cells. Liver enzyme levels are within normal limits. Which of the following is the most likely cause of the symptoms in this patient? A. Acute liver failure B. G6PD deficiency C. Malaria D. Sickle cell anemia E. Vitamin B12 deficiency anemia

Bitemproal hemianopsia **loss of peripheral vision in the temporal halves of each eye caused by a pituitary macroadenoma (prolactin-secreting adenoma)

A 29-year-old man comes to the clinic because he has had worsening headaches, weight gain, erectile dysfunction, decreased libido, and the development of breast tissue over the past year. He describes the headache as squeezing but not severe. He says that he and his partner have been trying to conceive for the past two years, but they have been unsuccessful. The patient recently broke his arm while throwing a baseball. Medical history is otherwise unremarkable, and he takes no medications. He does not smoke cigarettes or use illicit drugs; he drinks alcoholic beverages occasionally. Which of the following findings on physical examination is most likely? A. Binocular horizontal diplopia B. Bitemporal hemianopsia C. Hypertropia with upward drift D. Ptosis with ocular deviation to the outside and down E. Unilateral vision loss

Doxycycline ** Rocky Mountain spotted fever

A 29-year-old woman comes to the clinic because she has felt feverish for the past two days. She says she also has had a rash that started as little red bumps on her wrists and ankles and has now turned into little red dots all over her body. The rash started seven days after she got back from a hunting trip in Virginia. The patient does not remember getting bitten by any bugs while on the trip but is not sure. She is in a monogamous relationship. Temperature is 38.3°C (101.0°F). Physical examination shows lymphadenopathy in the axilla, cervical chain, and groin. A generalized maculopapular rash is noted with areas that appear to have transitioned to petechia. Which of the following is the most appropriate therapeutic agent? A. Aceteminophen B. Amoxicillin-clavulanic acid C. Doxycycline D. Prednisone E. Triamcinolone cream

Oxygen 12 L/min by nonrebreather mask **he has a cluster headache

A 30-year-old man comes to the clinic because he has had headaches during the past week. He describes the pain as searing and stabbing behind his right eye, and he rates the pain as 10 on a 10-point scale. He says the pain lasts about 30 minutes, resolves spontaneously, and then comes back a few hours later. He has had approximately four to five episodes per day that are accompanied by a red eye and tearing. The patient says he feels like he needs to move his body when the pain starts, and the pain is so severe that he has considered suicide. He has not had nausea and vomiting. The patient has no history of similar headaches. Medical history includes sleep apnea for which he is noncompliant with use of a continuous positive airway pressure machine. He smokes one pack of cigarettes per day. He says he stopped drinking because it seemed to make his symptoms worse; he does not use illicit drugs. Temperature is 37.0°C (98.6°), pulse rate is 88/min, and blood pressure is 120/80 mmHg. Oxygen saturation is 99%. Neurologic examination shows no focal deficits. Which of the following is the most appropriate treatment? A. Intravenous acetazolamide 500 mg B. Intravenous alteplase 100 mg C. Oral fluoxetine 20 mg D. Oral meloxicam 15 mg E. Oxygen 12 L/min by nonrebreather mask

McBurney sign **acute appendicitis

A 30-year-old man comes to the emergency department because he has had pain in the right lower abdomen for the past 12 hours. He says the symptoms came on suddenly and have been constant and are getting worse. He also has had fever, chills, and loss of appetite. Temperature is 38.9°C (102.1°F), pulse rate is 115/min, and blood pressure is 150/88 mmHg. Emergent CT scan shows stranding and inflammation around the cecum. Which of the following additional physical examination findings is most likely to be positive? A. Chandelier sign B. Cullen sign C. Grey-Turner sign D. McBurney sign E. Murphy sign

Hypothalamus **hyperthyroidism from the hypothalamus producing thyrotropin-releasing hormone (TRH)

A 30-year-old woman comes to the clinic because she has had a 15-lb unintentional weight loss over the past two months despite having an increased appetite. She also has had recent onset of tremors. Temperature is 37.0°C (98.6°F), pulse rate is 105/min, respirations are 16/min, and blood pressure is 121/62 mmHg. Oxygen saturation is 100%. On physical examination, a goiter is noted. Which of the following portions of the brain is most likely responsible for the initial regulation of the current hormone imbalance in this patient? A. Cerebellum B. Cerebral cortex C. Hypothalamus D. Pineal gland E. Pituitary gland

Initiate therapy with amphotericin B and flucytosine ** disseminated cryptococcal infection from bird droppings

A 31-year-old man is admitted to the hospital because he has altered mental status and signs of infection. He was initially evaluated because he had headache, fevers, neck stiffness, nausea, fatigue, and worsening confusion over the past week. Medical history includes herpes simplex virus and HIV. He says he has not been taking his antiretroviral medications. The patient has been taking vancomycin and piperacillin/tazobactam for the past three days. He works at a pet store that sells birds, and he denies any recent travel. Current temperature is 38.6°C (101.5°F), pulse rate is 110/min, and blood pressure is 130/102 mmHg. White blood cell count is 21,000/mm³. Lumbar puncture shows elevated pressure with cerebral spinal fluid showing decreased white blood cell count, decreased glucose level, and elevated protein level. Initial blood culture shows a facultative intracellular organism consistent with a Cryptococcus species. Which of the following is the most appropriate next step in treatment? A. Continue the current therapy with piperacillin/tazobactam and vancomycin B. Initiate therapy with acyclovir and prednisolone C. Initiate therapy with amphotericin B and flucytosine D. Initiate therapy with nystatin and terbinafine E. Switch to therapy with ceftriaxone and metronidazole

Interstitial markings ** acute bronchitis

A 32-year-old man comes to the clinic because he has had sore throat and cough for the past two days. He has not had hemoptysis, but the cough is productive of clear sputum. The patient has not had fever, shortness of breath, or difficulty breathing. He says he has a history of similar symptoms about once a year. Over-the-counter medications are giving good relief of symptoms. He smokes about one pack of cigarettes per day, drinks alcoholic beverages daily, and uses marijuana. Body mass index is 25 kg/m². Temperature is 37.0°C (98.6°F), pulse rate is 78/min, respirations are 12/min, and blood pressure is 118/68 mmHg. Oxygen saturation is 99% on room air. On physical examination, no abnormalities of the mucous membranes are noted. Auscultation of the lungs shows slight wheezing and rhonchi, which both clear after coughing. No egophony or E to A changes are noted. Results of complete blood cell count and basic metabolic panel are within normal limits. Which of the following findings is most likely on chest x-ray study? A. Interstitial markings B. Lobar infiltrates C. Perihilar infiltrates D. Pleural effusion E. Upper lobe cavitary lesions

Measurement of CT scan of chest **evaluate for carcinoid lung mass

A 32-year-old woman comes to the clinic because she has had slight shortness of breath with cough that is productive of slight, bloody sputum for the past week. She also has had facial flushing, weight gain, acne, and facial hair growth during the past two months. She says she does not feel ill, and she has no history of symptoms, asthma, or other lung conditions. The patient has never smoked cigarettes, drinks alcoholic beverages socially, and does not use illicit drugs. Her most recent menstrual period was two weeks ago, and she is not sexually active. Body mass index is 20 kg/m². Temperature is 36.0°C (97.6°F), pulse rate is 98/min, respirations are 12/min, and blood pressure is 178/104 mmHg. Oxygen saturation is 97%. On physical examination, the lungs are clear to auscultation. Coarse dark hair is noted on the upper lip with scattered comedones on the chin. Which of the following studies is the most appropriate next step in diagnosis? A. Complete blood cell count with differential B. CT scan of the chest C. Measurement of testosterone level D. Pulmonary function tests E. Vaginal ultrasonography

Holosystolic murmur best heard at the apex

A 32-year-old woman comes to the internal medicine clinic because she has had palpitations during the past year. Medical history includes group A streptococcal pharyngitis at 8 years of age. She says she was suspected of having rheumatic fever at that time and was initially treated with antibiotics, but she did not return for follow-up. Based on these findings, which of the following is most likely to be heard on cardiac auscultation? A. Crescendo-decrescendo systolic murmur heard best at the right upper sternal border B. Diastolic decrescendo blowing murmur heard best at the left upper sternal border C. Harsh mid systolic ejection crescendo-decrescendo murmur heard best at the left upper sternal border D. Holosystolic blowing high-pitched murmur heard best at the left mid sternal border E. Holosystolic murmur heard best at the apex

Propylthiouracil **thyroid storm likely precipitated by surgery

A 32-year-old woman who is being treated in the hospital after undergoing laparoscopic cholecystectomy two days ago has had altered mental status for the past 30 minutes. Medical history includes well-controlled Graves disease. Pulse rate is 148/min, respirations are 22/min, and blood pressure is 165/110 mmHg. Oxygen saturation is 95%. Laboratory studies obtained this morning show white blood cell count of 7000/mm³, thyroid-stimulating hormone level of .018 µU/L, free thyroxine level of 3.5 µg/dL, and glucose level of 98 mg/dL. In addition to a beta-blocker, which of the following medications is most appropriate to administer to this patient? A. Ampicillin/sulbactam B. Aspirin C. Insulin glargine D. Levothyroxine E. Propylthiouracil

Renal ultrasonography **polycystic kidney disease

A 33-year-old man comes to the internal medicine clinic because he has had dark urine and intermittent abdominal pain during the past week. Medical history includes no chronic disease conditions, and he has no history of recent trauma. He does not know his family's past medical history well, though he thinks that his mother had some kidney and liver issues. Temperature is 36.7°C (98.0°F), pulse rate is 80/min, respirations are 16/min, and blood pressure is 168/102 mmHg. Oxygen saturation is 98% on room air. Physical examination shows a late systolic crescendo murmur with a mid systolic click. Abdominal examination shows bilateral masses that are tender to palpation. Results of urinalysis are positive for red blood cells, negative for white blood cells, negative for casts, and negative for nitrites. Which of the following diagnostic modalities is most appropriate to use to screen members of this patient's family to assess if they are affected by the same condition? A. CT scan of the abdomen B. Cystoscopy C. Echocardiography D. Genetic sequencing E. Renal ultrasonography

24-hour ambulatory cardiac monitoring

A 33-year-old man comes to the internal medicine clinic because he has had several intermittent episodes of "heart flutters" during the past month. He says the episodes last for 10 to 30 seconds and resolve without intervention; he has had several over the past few days. He does not have any associated chest pain or dizziness. Temperature is 37.0°C (98.6°F), pulse rate is 72/min, respirations are 16/min, and blood pressure is 124/83 mmHg. Electrocardiography shows a normal sinus rhythm. Which of the following studies is the most appropriate next step? A. 24-Hour ambulatory cardiac monitoring B. Cardiac catheterization C. Cardiac nuclear scanning D. Cardiac stress testing E. Echocardiography

Biguanides **Metformin is the 1st line drug

A 33-year-old transgender man with diabetes mellitus comes to the internal medicine clinic for follow-up to discuss recent laboratory results. Laboratory studies show a fasting plasma glucose level of >126 mg/dL on two occasions and hemoglobin A1c of 7.2%. He has tried diet and exercise changes with limited success. Medication needs to be initiated, and a first-line oral antihyperglycemic agent is chosen. This medication decreases hepatic glucose production as well as intestinal absorption while increasing the body's sensitivity to insulin, and most likely belongs to which of the following classes of antihyperglycemic agents? A. Biguanides B. Glucagon-like peptide 1 receptor agonists C. Meglitinides D. Sulfonylureas E. Thiazolidinediones

Do you have a family history of similar symptoms? **Huntington disease which is an autosomal dominant disorder characterized by motor, cognitive, and psychiatric dysfunction with chorealike movements

A 34-year-old woman comes to the clinic because she has had rhythmic and involuntary movement of the arms during the past year. She says the symptoms started with involuntary finger movements that then progressed to involve her wrists and forearms. Over the past two years, the patient has had irritability and impulsivity, which prompted her to be evaluated for possible bipolar disorder. She says she used to be very organized but has been scattered and has had a hard time concentrating over the past two years. Temperature is 37.1°C (98.8°F), pulse rate is 80/min, respirations are 16/min, and blood pressure is 128/87 mmHg. Oxygen saturation is 98% on room air. Which of the following is the most important question to ask this patient? A. Do you have a family history of similar symptoms? B. Do you have a history of brain trauma? C. Do you have a history of recent travel? D. Do you have a history of smoking cigarettes, drinking alcoholic beverages, or using illicit drugs? E. Do you have a medical history of chronic disease conditions?

Iron deficiency anemia

A 34-year-old woman comes to the clinic because she has had significant fatigue and has been having a hard time getting out of bed in the morning for the past two months. She says she has never had this symptom before and is very healthy because she exercises regularly and eats a healthy diet. The patient has not had weight loss or changes in lifestyle; however, she says she has been under a lot of stress at home and at work. Her only medication is an occasional acetaminophen for headaches. Vital signs are within normal limits, and physical examination shows no abnormalities. Laboratory findings include the following: thyroid-stimulating hormone 1.3 µU/mL; white blood cell count 8600/mm³; hemoglobin level 10.5 g/dL; hematocrit 34.5%; platelet count 250,000/mm³; mean corpuscular volume 73 µm³; mean corpuscular hemoglobin level 26 pg/cell; mean corpuscular hemoglobin concentration 26 g/dL; and red cell distribution width 20%. Result of serum beta-human chorionic gonadotropin test is negative. Which if the following is the most likely cause of the findings in this patient? A. Depression B. Hypothyroidism C. Iron deficiency anemia D. Pregnancy E. Vitamin B12 deficiency

Erythropoietin deficiency ** anemia of chronic kidney disease which is a form of normocytic, normochromic hypo proliferative anemia and is due to decreased production of erythropoietin by the kidneys

A 34-year-old woman with type 1 diabetes mellitus comes to the internal medicine clinic for routine follow-up. The patient has no concerns. Medical history includes worsening kidney disease over the past three years. Her most recent estimated glomerular filtration rate is 35 mg/mL, and she is not yet on dialysis. Physical examination shows no abnormalities. Recent laboratory studies show hemoglobin level of 8.5 g/dL, hematocrit of 27%, and mean corpuscular volume of 92 μm³. Peripheral blood smear shows normocytic, normochromic red blood cells. Which of the following is the most likely explanation for the anemia in this patient? A. Acute blood loss B. Erythropoietin deficiency C. Iron deficiency anemia D. Polycythemia vera E. Sideroblastic anemia

Topical metronidazole **papulopustular rosacea

A 36-year-old woman comes to the office because she has had intermittent acne-like breakouts on her cheeks and chin during the past eight months. She says that she has had flushing of the skin as well as burning and stinging in these same areas for years. Temperature is 37.1°C (98.8°F), pulse rate is 80/min, respirations are 16/min, and blood pressure is 128/87 mmHg. Oxygen saturation is 98% on room air. Physical examination shows erythema, telangiectasias, papules, and tiny pustules on both cheeks and chin. Which of the following is the most appropriate initial treatment? A. Laser therapy B. Oral erythromycin C. Oral isotretinoin D. Topical fluorinated corticosteroids E. Topical metronidazole

Lactated ringer solution **acute pancreatitis

A 38-year-old white woman, gravida 3, para 3, is admitted to the hospital for treatment of upper abdominal pain that is severe and radiates to her back. She also has nausea and vomiting and cannot keep anything down. The patient has had similar symptoms in the past but never this bad. She says that nothing she has done has helped relieve her symptoms. She does not take any medications and uses an intrauterine device for birth control. The patient smokes one-half pack of cigarettes daily and admits to drinking heavily over the past six days. She typically drinks one to two pints of hard liquor daily. Body mass index is 36 kg/m². Temperature is 37.0°C (98.6°F), pulse rate is 118/min, and blood pressure is 110/86 mmHg. Physical examination shows tenderness to palpation in the epigastric region. Laboratory studies show the following: white blood cell count 13,000/mm³; hemoglobin and hematocrit levels within normal limits; alkaline phosphatase 50 U/L; creatinine 1.1 mg/dL; aspartate aminotransferase 305 U/L; alanine aminotransferase 150 U/L; and lipase 710 U/L. Troponin is undetectable at presentation as well as six hours later. Result of quantitative beta-human chorionic gonadotropin test is negative. Test of the stool for blood is negative. Abdominal CT scan and ultrasonography show no abnormalities. Which of the following is the most appropriate initial management? A. Cholecystectomy B. Endoscopy C. Lactated Ringer solution D. Pantoprazole therapy E. Piperacillin/tazobactam

Measurement of vitamin B12 level

A 38-year-old woman comes to the clinic because she has had worsening fatigue over the past six months, which is making it hard for her to take care of her children. She says that she has not made any lifestyle changes and continues to exercise three times a week, although her fatigue has made this difficult. She has been eating a vegan diet for the past five years. Her weight has remained stable, and she has no other signs or symptoms. The patient has never smoked, rarely drinks alcoholic beverages, and does not use illicit drugs. Medical history includes no chronic disease conditions, and she takes no medications. She underwent gastric bypass surgery 10 years ago. Body mass index is 21 kg/m². Pulse rate is 102/min, and blood pressure is 102/68 mmHg. The patient does not appear to be in acute distress. On physical examination, slight jaundice is noted. Palpation of the thyroid gland shows normal size and no masses. Cardiopulmonary and abdominal examinations show no abnormalities. Complete blood cell count shows the following: white blood cell count 10/mm³; hemoglobin 8.1 g/dL; hematocrit 21%; mean corpuscular volume 106 µm³; mean corpuscular hemoglobin 29 pg/cell; mean corpuscular hemoglobin concentration 34 g/dL; and platelet count 200/mm³. Which of the following additional laboratory studies is most likely to be abnormal? A. Hepatitis C antibody test B. Iron panel C. Measurement of riboflavin level D. Measurement of thyroid-stimulating hormone level E. Measurement of vitamin B12 level

Erythropoietin **induces reticulocyte release and helps with anemia that results from end-stage renal disease

A 38-year-old woman with a 20-year history of type 1 diabetes mellitus comes to the internal medicine clinic for follow-up examination. Medical history also includes end-stage renal disease, for which she undergoes hemodialysis. As part of her care plan, the patient is administered regular injections of a drug that aids in inducing reticulocyte release from the bone marrow. Which of the following drugs is this patient most likely being administered? A. Erythropoietin B. Folic acid C. Infliximab D. Platelet-derived growth factor E. Vitamin B12

Age 50

A 39-year-old man comes to the internal medicine clinic for routine follow-up of well-controlled hypertension and diabetes mellitus. He is about to turn 40 and is a bit anxious about getting older. During the interview, he asks about screening for prostate cancer. Family medical history does not include prostate cancer. Despite being informed about the uncertainties and potential risks surrounding this testing, the patient still wants to be screened for prostate cancer. It is most appropriate to recommend to this patient that a conversation about prostate cancer screening should begin at which of the following ages? A. Age 40 B. Age 45 C. Age 50 D. Age 60 E. Current age

Pyridostigmine **myasthenia gravis treated with acetylcholinesterase inhibitor

A 39-year-old woman comes to the clinic because she has had weakness on and off for the past eight months since she delivered her last child. The patient says that her eyes feel tired and she has double vision in the afternoon. She also has had a harder time eating dinner because her jaw gets tired, and she has to focus on eating otherwise she sometimes chokes. Her legs have been very weak recently, and she has to rest when she climbs the stairs at her apartment when she comes home from work at the end of the day. The symptoms are relieved by rest and are not present when she wakes up in the morning. She does not have difficulty breathing or bowel or bladder symptoms, and she has not noticed numbness or tingling. She has noticed that hot showers and temperatures affect her symptoms. The patient does not smoke cigarettes, drink alcoholic beverages, or use illicit drugs. Temperature is 37.0°C (98.6°F), pulse rate is 78/min, respirations are 12/min, and blood pressure is 130/78 mmHg. Oxygen saturation is 97%. On physical examination, the pupils are equal and reactive to light. Skin sensation is intact. Muscle strength is equal and 5/5. Which of the following medications is the most appropriate initial treatment? A. Acetazolamide B. Alteplase C. Cefepime D. Glatiramer E. Pyridostigmine

Inflammatory bowel disease ** IBD are reasons to start colon cancer screening early

A 40-year-old man comes to the clinic for routine physical examination. During the interview, he asks if he needs to get a colonoscopy. It is most appropriate to tell this patient that he will only need a colonoscopy at this age if he has which of the following findings on history and physical examination? A. A 20—pack-year history of cigarette smoking B. African American ethnicity C. Inflammatory bowel disease D. Irritable bowel syndrome E. Significant diverticulosis

Prednisolone **severe acute alcoholic hepatitis should be treated with IV fluids and steroids such as prednisolone

A 41-year-old man comes to the emergency department (ED) because he has had worsening abdominal pain, subjective fevers, and poor appetite over the past week. He is also very confused, is slurring his speech, and falls asleep multiple times during the ED intake. His partner says that he seems like he has a suntan, but he has not spent any time in the sun. The patient has not had similar symptoms in the past. Medical history is unremarkable, and he takes no medications. When he comes to, he says he usually drinks one to two pints of hard liquor daily but recently has been stressed and has increased to almost a fifth of hard liquor daily. His most recent alcoholic drink was just before coming to the ED. He has no history of intravenous drug use. Temperature is 37.2°C (98.9°F). The patient appears thin. Physical examination shows generalized jaundice including the conjunctiva. Tenderness to palpation is noted in the right upper quadrant, and the liver is protuberant. No ascites is noted. The patient seems extremely lethargic. Laboratory studies show the following: white blood cell count 13,100/mm³ with 90% neutrophils; aspartate aminotransferase 280 U/L; alanine aminotransferase 140 U/L; bilirubin 14.5 mg/dL; gamma-glutamyl transferase 290 U/L; and international normalized ratio 2.1. CT scan of the abdomen shows fatty changes to the liver without other abnormalities. Along with supportive IV fluids, which of the following is the most appropriate initial therapy? A. Ceftriaxone B. Intravenous immunoglobulin C. Ledipasvir/sofosbuvir D. Piperacillin/tazobactam E. Prednisolone

Screen at age 45 years regardless of symptoms **ADA guidelines suggest all adults > 45 should be screened every 3 years

A 41-year-old man comes to the internal medicine clinic for a routine physical examination. He says he is active and healthy. His medical history includes eczema, and he does not take any medications. Body mass index is 22 kg/m². Pulse rate is 65/min, respirations are 16/min, and blood pressure is 126/72 mmHg. The patient appears athletic and well. During the interview, he says that a friend of his was recently diagnosed with diabetes mellitus, and the patient asks if he should be screened. Which of the following indications is the most appropriate related to diabetes screening in this patient? A. Only screen if BMI >/=25 kg/m² B. Only screen if symptomatic C. Only screen if there are risk factors D. Screen at age 45 years regardless of symptoms E. Screen at age 55 years regardless of symptoms

ultrasonography

A 41-year-old patient who identifies as gender nonconforming comes to the clinic because of intermittent pain in the right upper quadrant that has been worsening over the past two weeks. Today, the patient feels "pretty awful." The pain radiates to the shoulder blade on the right side and is worse in the evenings, particularly if they eat fatty foods. Body mass index is 32 kg/m², but they have been on a weight loss program and has lost 15 lb over the past three months. Temperature is 38.3°C (101.0°F). Physical examination shows tenderness with palpation to the right upper quadrant. White blood cell count is elevated at 12,000/mm³. Which of the following abdominal imaging studies is the most appropriate next step in diagnosis? A. CT scan B. MR cholangiopancreatography C. MRI D. Ultrasonography E. X-ray study

Elevated bicarbonate level **patient is suffering from obesity hypoventilation syndrome

A 43-year-old man comes to the clinic because he has had worsening fatigue and shortness of breath over the past six weeks. The shortness of breath is present even with minimal exertion. He says his partner is concerned because he occasionally stops breathing at night when he is snoring. These symptoms came on gradually and have been making it hard for him to function in activities of normal life. Medical history includes prediabetes and high blood pressure for which he takes amlodipine. He does not smoke cigarettes, drink alcoholic beverages, or use illicit drugs. Body mass index is 42 kg/m². Temperature is 37.3°C (99.2°F), pulse rate is 102/min, respirations are 22/min, and blood pressure is 128/78 mmHg. Oxygen saturation is 94%. Physical examination shows a thick neck with a crowded oropharynx. Cardiac examination shows a regular rate and a prominent S2. The lungs are clear to auscultation. Bilateral +2 pitting edema is noted in the lower extremities. Which of the following abnormal laboratory results is most likely? A. Decreased blood urea nitrogen level B. Decreased sodium level C. Elevated bicarbonate level D. Elevated creatinine level E. Elevated potassium level

Costovertebral tenderness

A 43-year-old man comes to the emergency department because he had acute onset of severe, colicky pain in the left flank four hours ago. He says the pain radiates to his left groin. Medical history includes hypertension treated with hydrochlorothiazide. Urinalysis shows microscopic hematuria. Noncontrast CT scan shows an 8-mm stone at the ureteropelvic junction. Intravenous fluids and analgesics are administered. Results of extracorporeal shock wave lithotripsy are pending. Which of the following findings on physical examination is most likely? A. Absent cremasteric reflex B. Costovertebral angle tenderness C. Decrease in pain with elevation of the left testicle D. Tenderness on digital rectal examination E. Tenderness to palpation in the right upper quadrant

Sodium

A 43-year-old man comes to the internal medicine clinic for routine physical examination. He is asymptomatic, and medical history does not include any chronic disease conditions. Family medical history includes hypertension in his father, brother, and sister. The patient's blood pressure is 138/85 mmHg. Physical examination shows no abnormalities. Decreased intake of which of the following substances is most likely to decrease this patient's risk of hypertension? A. High-fructose corn syrup B. Potassium C. Refined carbohydrates D. Saturated fats E. Sodium

Thyroid-stimulating hormone level

A 45-year-old patient who identifies as genderqueer comes to the clinic because of a myriad of symptoms over the past two months. During this time, they have gained 10 lb despite no changes in diet or exercise. In addition, the patient reports fatigue, difficulty concentrating at work, hair loss, and constipation. Pulse rate is 58/min. Physical examination shows goiter. The most appropriate next step in diagnosis is measurement of which of the following laboratory values? A. Free thyroxine index B. Free thyroxine level C. Free triiodothyronine level D. Thyroid antibodies E. Thyroid-stimulating hormone level

Bacterial infection

A 45-year-old woman is being treated in the hospital for respiratory distress. Her symptoms started 48 hours ago with cough and fevers. She takes no medications. She has a 20—pack-year history of cigarette smoking. Temperature is 38.9°C (102.0°F), and pulse rate is 130/min. White blood cell count is 21/mm³, neutrophils are 90%, lymphocytes are 15%, monocytes are 4%, eosinophils are 3%, and immature granulocytes are 3%. Toxic granulocytes are noted. Which of the following is the most likely explanation for these findings? A. Acute leukemia B. Bacterial infection C. Corticosteroid use D. Parasitic infection E. Viral infection

Hydrochlorothiazide **HCTZ is a common diuretic with side effects of hypercalcemia, hyponatremia, hypokalemia, hyperuricemia, and hyperglycemia

A 46-year-old patient comes to the internal medicine clinic for follow-up two months after initiating treatment for hypertension. They have been tolerating the medication well. Pulse rate is 70/min, respirations are 18/min, and blood pressure is 125/80 mmHg. Laboratory studies show a slightly elevated calcium level. Which of the following medications most likely caused this abnormality? A. Amlodipine B. Furosemide C. Hydrochlorothiazide D. Propranolol E. Spironolactone

Staphylococcus aureus **septic arthritis from IV drugs

A 48-year-old man comes to the emergency department because he has had worsening pain in the left knee over the past eight days. He says he has a history of intravenous drug use. Temperature is 39.4°C (103.0°F); all other vital signs are within normal limits. On physical examination, the left knee is erythematous. Edema and tenderness to palpation are noted. There is decreased range of motion of the knee, and the patient is unable to bear weight on the leg. Laboratory studies show elevated C-reactive protein level and increased erythrocyte sedimentation rate. Arthrocentesis shows a white blood cell count of 70,000/mm³, and gram-positive organisms are present. Which of the following is the most likely causal organism? A. Group B Streptococcus B. Haemophilus influenzae C. Neisseria gonorrhoeae D. Pseudomonas aeruginosa E. Staphylococcus aureus

Low-dose CT scan of the chest ** test of choice for 50-80 years of age with 20-pack year history of smoking that still smokes or quit within the past 15 years

A 51-year-old woman comes to the clinic for routine physical examination. She has no symptoms and has been feeling well. Medical history includes no chronic disease conditions, and she takes no medications. Family history includes breast cancer in her mother and heart disease in her father. She smoked one pack of cigarettes per day for 20 years, but she quit five years ago. She drinks two alcoholic beverages per week and does not use illicit drugs. Temperature is 37.0°C (98.6°F), pulse rate is 90/min, respirations are 12/min, and blood pressure is 128/80 mmHg. Oxygen saturation is 92% on room air. On physical examination, the lungs are clear to auscultation. No peripheral edema is noted. To screen this patient for lung cancer, which of the following studies is most appropriate? A. Chest x-ray study B. Low-dose CT scan of the chest C. MRI of the chest D. Pulmonary function testing E. Spiral CT scan of the chest

Elevated baseline ACTH; cortisol suppression noted **high-dose dexamethasone suppression test differentiates Cushing disease from ectopic ACTH production; Cushing syndrome/disease that is specifically caused by an ACTH-secreting pituitary adenoma supported by cortisol suppression

A 52-year-old woman comes to the internal medicine clinic because she has had a 15-lb weight gain over the past two months despite no change in appetite or diet. She says that the fat is primarily in her belly region, while her legs and arms have stayed relatively slim. The patient says she has abnormal stretch marks around her abdomen. Blood pressure is 145/85 mmHg. Physical examination shows truncal obesity, fat deposition at the base of the neck, and coarse hair growth on the face. Based on the results of a 24-hour urine free cortisol test, a pituitary adenoma is suspected. Baseline adrenocorticotropic hormone (ACTH) levels and a high-dose dexamethasone suppression test are ordered. Which of the following findings on these tests is most likely to confirm this diagnosis? A. Decreased baseline ACTH level; cortisol suppression noted B. Decreased baseline ACTH level; no cortisol suppression noted C. Elevated baseline ACTH level; cortisol suppression noted D. Elevated baseline ACTH level; elevated cortisol level above pretest levels E. Elevated baseline ACTH level; no cortisol suppression noted

Electrocardiography **this is the most appropriate initial step in diagnosing chest pain

A 53-year-old man comes to the internal medicine clinic because he has had persistent chest pressure and a bloating sensation in his epigastrium for the past hour. Medical history includes peptic ulcer disease, and he has no significant cardiac history. Pulse rate is 76/min, respirations are 18/min, and blood pressure is 145/80 mmHg. Oxygen saturation is 98% on room air. Physical examination shows no abnormalities. Which of the following is the most appropriate initial step in diagnosis? A. Echocardiography B. Electrocardiography C. Esophagogastroduodenoscopy D. Fecal occult blood test E. X-ray studies of the upper gastrointestinal tract

Hypocalcemia

A 53-year-old woman is recovering in the hospital after undergoing thyroidectomy two days ago for treatment of Graves disease. Currently, she says she has anxiety, muscle cramps, and tingling in her hands, feet, and around her mouth. On physical examination, light tapping of the region anterior to the ears causes twitching of the perioral muscles. Which of the following is the most likely explanation for this physical examination finding? A. Hyperkalemia B. Hypermagnesemia C. Hypocalcemia D. Hyponatremia E. Hypophosphatemia

Vancomycin **green diarrhea and recent antibiotic use is warranted by C diff.

A 54-year-old woman comes to the clinic because she has had worsening generalized abdominal cramping with severe watery diarrhea for the past 24 hours. She says the diarrhea is fowl-smelling and green in color; she has not noticed blood in the stool. The patient says she completed a course of antibiotics 17 days ago. She has no history of recent travel or of drinking untreated water. Temperature is 38.2°C (100.8°F), pulse rate is 100/min, and blood pressure is 110/68 mmHg. The patient appears ill but not toxic. Results of urinalysis are within normal limits. A stool sample is collected for additional testing. Which of the following is the most appropriate initial drug therapy? A. Amoxicillin-clavulanic acid B. Clindamycin C. Loperamide D. Prednisone E. Vancomycin

Sarcoidosis **noncaseating granulomas and hilarious adenopathy

A 55-year-old man comes to the clinic for follow-up after a recent hospitalization for treatment of worsening shortness of breath. He says he has had chronic cough and shortness of breath over the past few years, but these symptoms became worse a week ago. While the patient was in the hospital, chest x-ray study showed bilateral hilar adenopathy with reticular opacities, and bronchoscopy with a biopsy showed noncaseating granulomas. Currently, he says that he continues to have worsening shortness of breath and cough. Medical history includes painful lesions on his shins, and intermittent painful and photosensitive episodes of his eyes. The patient does not smoke cigarettes, drink alcoholic beverages, or use illicit drugs. Temperature is 36.6°C (97.9°F), pulse rate is 80/min, respirations are 12/min, and blood pressure is 122/88 mmHg. Oxygen saturation is 94% on 2 L of oxygen via nasal cannula. The patient does not appear to be in acute distress. On physical examination, auscultation of the lungs shows scattered expiratory crackles. No peripheral edema is noted. Tender round nodules are noted on the shins. Which of the following is the most likely diagnosis? A. Asthma B. Chronic obstructive pulmonary disease C. Congestive heart failure D. Pneumonia E. Sarcoidosis

Bell palsy

A 55-year-old man comes to the emergency department because he has had worsening facial droop since he awoke 6 hours ago. His spouse noticed that the right side of his face was drooping when he came down for breakfast. He says he is having a hard time drinking and talking, and he is struggling to close his eye on the right side. He does not have any pain or discomfort. Medical history includes poorly controlled diabetes mellitus. The patient does not drink alcoholic beverages, smoke cigarettes, or use illegal drugs. He has no history of recent travel or insect bites. Pulse rate is 88 /min, and blood pressure is 130/82 mmHg. Physical examination shows marked flaccid weakness of the muscles on the right side of the face including the forehead. Examination of the ears and skin show no abnormalities. Which of the following is the most likely diagnosis? A. Bell palsy B. Cerebrovascular accident C. Glioblastoma multiforme D. Lyme disease E. Ramsay Hunt syndrome

Phosphodiesterase-5 inhibitors **sildenafil and tadalafil for ED

A 55-year-old man comes to the internal medicine clinic because he has had difficulty getting an erection during the past six months. He says that he no longer gets morning or nighttime erections. The patient has not had depressive symptoms or any recent life stressors. Medical history includes diabetes mellitus that is well-controlled with metformin. Body mass index is 34 kg/m². Laboratory studies, including complete blood cell count, complete metabolic panel, lipid panel, thyroid-stimulating hormone level, and morning testosterone level are all within normal limits. In addition to lifestyle changes, it is most appropriate to recommend that this patient begin therapy with which of the following classes of medications? A. 5-Alpha-reductase inhibitors B. Alpha-blockers C. Angiotensin-converting enzyme inhibitors D. Phosphodiesterase-5 inhibitors E. Synthetic antidiuretic hormones

Acute pericarditis

A 55-year-old man with hypertension and hyperlipidemia is in the hospital after undergoing coronary artery bypass grafting two days ago for an acute myocardial infarction. The chest pain resolved after surgery, but the patient suddenly has onset of acute chest pain that is sharp and pleuritic; he says that the pain improves with sitting up and leaning forward. He is afebrile; pulse rate is 80/min, and respirations are 18/min. Oxygen saturation is 95% on room air. On physical examination, auscultation of the heart shows a superficial scratchy sound. Result of troponin test is negative. Electrocardiography shows diffuse ST elevation. Echocardiography shows no evidence of pericardial effusion. Which of the following is the most likely diagnosis? A. Acute pericarditis B. Acute ST-elevation myocardial infarction C. Left ventricular aneurysm D. Papillary muscle rupture E. Postmyocardial infarction syndrome

Diffuse esophageal spasm **dysphagia and the sensation of food getting stuck relieved by nitroglycerin

A 55-year-old pateint who identifies as genderqueer is admitted to the inpatient service for evaluation of chest pain. They have had slow progression of substernal chest pain over the past two months, and the symptoms have worsened over the past two days. They describe the pain as squeezing, and it is made much worse with eating or drinking, particularly hot liquids. The patient feels like food and liquids are getting stuck in the chest, and they have had one episode of regurgitation. Discomfort is not associated with exertion. They have not had any shortness of breath or blood in their stool. Temperature is 37.1°C (98.8°F), pulse rate is 80/min, respirations are 16/min, and blood pressure is 128/87 mmHg. Oxygen saturation is 98% on room air. Physical examination shows no abnormalities. On laboratory studies, troponin is undetectable upon presentation and also undetectable at 6 hours. Results of complete blood cell count and complete metabolic panel are within normal limits. Electrocardiography obtained while the patient is having symptoms shows a normal sinus rhythm without ischemia or ectopy. Chest x-ray study shows no abnormalities. Nitroglycerin is administered and improves the patient's discomfort from an 8/10 to a 3/10. Which of the following is the most likely diagnosis? A. Acute coronary syndrome B. Diffuse esophageal spasm C. Gastroesophageal reflux disease D. Small-cell carcinoma of the lung E. Unstable angina

Chronic bronchiectasis **likely caused by lifelong infection of RA and Sjogren syndrome

A 55-year-old woman comes to the clinic because she has had worsening shortness of breath and cough over the past few years. She says she normally gets better after treatment, but she has been coughing up clear mucus, has had shortness of breath, and has had some slight pain in the chest wall over the past six months. Recent results of testing for asthma and heart conditions were negative. Medical history includes rheumatoid arthritis and Sjögren syndrome. She has never smoked cigarettes and does not drink alcoholic beverages or use illicit drugs. Temperature is 36.5°C (97.7°F), pulse rate is 90/min, respirations are 12/min. Oxygen saturation is 90% on room air. The patient does not appear to be in acute distress but coughs occasionally throughout the interview. On physical examination, cachexia is noted. Mucus membranes are pink, very dry, and without discharge. No cervical lymphadenopathy is noted. Auscultation of the lungs shows bilateral basilar crackles with rare wheezing. No peripheral edema is noted. The fingertips look like an upside down spoon. Chest x-ray study shows parallel linear densities with tram-track opacities and ring shadows; no infiltrates are noted. Which of the following is the most likely diagnosis? A. Acute bronchitis B. Acute pulmonary neoplasm C. Acute tuberculosis D. Chronic bronchiectasis E. Chronic pneumonia

Pericardiocentesis **cardiac tamponade

A 55-year-old woman is admitted to the hospital after she came to the emergency department because she had worsening dizziness and dyspnea on exertion for the past four hours. Medical history includes systemic lupus erythematosus. Initial vital signs include pulse rate of 118/min, respirations of 26/min, and blood pressure of 82/40 mmHg. Oxygen saturation was 88% on room air. Physical examination shows distant heart sounds and jugular venous distention. Blood pressure markedly decreases with inspiration. Electrocardiography shows low voltages and electrical alternans. The patient was initially alert and conversational in the emergency department; since admission, she has quickly become confused and has sudden loss of consciousness. Which of the following is the most appropriate next step to increase this patient's chance of survival? A. Administration of aspirin B. Administration of heparin C. Coronary reperfusion D. Insertion of a chest tube E. Pericardiocentesis

Colonoscopy **has subacute bacterial endocarditis from fever, Osler nodes, splinter hemorrhages, and Janeway lesions. Streptococcus Boris has a concern for colon cancer

A 57-year-old man is admitted to the hospital after he was evaluated in the emergency department because he had fever for the past four days. Temperature is 38.1°C (100.6°F). Physical examination shows nodules on the finger pads that are tender to palpation, splinter hemorrhages in the nailbeds, and painless erythematous macules on the palms and soles. Auscultation of the heart shows a new systolic murmur heard best at the apex. Echocardiography shows vegetations on the mitral valve. Results of blood cultures that were obtained more than 12 hours apart are positive for Streptococcus bovis. In addition to intravenous antibiotics, which of the following is the most appropriate next step in management? A. Anticoagulation B. Colonoscopy C. Esophagogastroduodenoscopy D. Pericardiocentesis E. Valve replacement

Stomach ulceration

A 58-year-old patient who identifies as non-binary is brought to the emergency department by ambulance after a coworker found them in the bathroom vomiting blood that day. En route to the hospital, the patient lost consciousness. The coworker recalls the patient having lower back pain and was taking naproxen and ibuprofen. Pulse rate is 130/min, respirations are 28/min, and blood pressure is 82/40 mmHg. The patient is immediately transferred to the operating room and undergoes upper endoscopy and is subsequently admitted to the ICU. Which of the following is the most likely explanation for the findings in this patient? A. Aortoenteric fistula B. Epistaxis from the sphenopalatine artery C. Esophageal varices D. Longitudinal mucosal ulcerations at the gastroesophageal junction E. Stomach ulceration

referral for surgery ** surgical management is recommended for thyroid follicular cancer lesions

A 58-year-old woman comes to the internal medicine clinic because she has had a lump on the anterior portion of her neck around the area of the thyroid gland for the past three months. She has no other symptoms. Medical history includes hypertension, for which she takes hydrochlorothiazide. Temperature is 37.1°C (98.8°F), pulse rate is 80/min, respirations are 16/min, and blood pressure is 128/87 mmHg. Oxygen saturation is 98% on room air. On physical examination, a smooth nodule can be felt on the right side of the thyroid gland, which stays fixed in place on swallowing. Laboratory studies show a thyroid-stimulating hormone level of 2.4 µU/L. The patient undergoes fine-needle aspiration biopsy and follicular adenoma is diagnosed. Which of the following is the most appropriate next step in management? A. Antibiotic therapy B. Medication management C. Observation with ultrasonography every six months D. Radioactive iodine therapy E. Referral for surgery

Atorvastatin

A 59-year-old man with diabetes mellitus and hypertension comes to the internal medicine clinic for routine follow-up examination. At his most recent visit, laboratory studies showed total cholesterol level of 220 mg/dL and high-density lipoprotein (HDL) cholesterol level of 30 mg/dL. His 10-year risk for atherosclerotic cardiovascular disease was calculated at 28.6%, and drug therapy was initiated. Current laboratory studies show total cholesterol level of 140 mg/dL and HDL cholesterol level of 50 mg/dL. Based on these findings, which of the following drugs has this patient most likely been taking? A. Atorvastatin B. Furosemide C. Gemfibrozil D. Hydrochlorothiazide E. Propranolol

Hypertrophy of the septal wall **hypertrophic cardiomyopathy has a murmur best heard at the lower left sternal border and is unique in that it increases in intensity with decreased. venous return (Valsalva, standing, exertion) and Echocardiography will show septal wall thickening and hypertrophy of the left ventricle

A 60-year-old man comes to the clinic because he has had dyspnea on exertion for the past three months. Temperature is 37.1°C (98.8°F), pulse rate is 80/min, respirations are 16/min, and blood pressure is 128/87 mmHg. Oxygen saturation is 98% on room air. Physical examination shows a harsh systolic crescendo-decrescendo murmur that increases in intensity with Valsalva maneuver. Electrocardiography shows left ventricular hypertrophy and Q waves. Which of the following findings is most likely on echocardiography? A. Ejection fraction of 25% B. Enlarged atria C. Enlarged left ventricle D. Hypertrophy of the septal wall E. Wall motion abnormality

Acyclovir **describes shingles or herpes zoster

A 60-year-old patient comes to the clinic because of spider bites on the chest. They have had burning, itchy pain on the right side of the chest for about 24 hours. This morning, they noticed redness and a couple of blisters. The patient does not remember seeing any spiders but says they have been working in the yard. The patient has not had fever, chills, or recent illness. Findings on physical examination are shown in the image. Which of the following is the most appropriate treatment? A. Acyclovir B. Aloe vera gel C. Cefalexin D. Ketoconazole E. Prednisone

Left anterior descending artery

A 61-year-old man comes to the emergency department because he has had crushing substernal chest pain radiating to his left arm for the past two hours. Medical history includes hypertension and hyperlipidemia. The patient has a history of smoking cigarettes. Temperature is 37.0°C (98.6°F), pulse rate is 94/min, respirations are 20/min, and blood pressure is 150/90 mmHg. Oxygen saturation is 95% on room air. The patient is diaphoretic and is clutching his upper left chest with his right arm. Cardiac troponin levels are significantly elevated. Electrocardiography shows ST-elevation in leads V1 through V4. The patient undergoes percutaneous coronary intervention and is admitted to the ICU. The most likely cause is occlusion of which of the following arteries? A. Circumflex artery B. Left anterior descending C. Left main coronary artery D. Posterior descending artery E. Right coronary artery

Administration of Fresh Frozen Plasma ** he has disseminated intravascular coagulation

A 61-year-old man comes to the emergency department because he has had worsening fevers, chills, cough, and shortness of breath over the past 10 days. He says his symptoms started to get better a few days ago but then acutely worsened. The patient noticed some blood in his stool this morning. Medical history includes chronic obstructive pulmonary disease and hypertension, for which he uses an albuterol inhaler and takes lisinopril, respectively. He quit smoking two years ago, occasionally drinks alcoholic beverages, and does not use illegal drugs. Temperature is 39.2°C (102.6°F), pulse rate is 123/min, respirations are 24/min, and blood pressure is 146/82 mmHg. Oxygen saturation is 92% on room air. The patient is not in distress and is protecting his airway. Physical examination shows slight bleeding around the nares and gums. Non-blanching, 1-mm red macules are noted on his body but worse on his legs. Auscultation of the lungs shows coarse lung sounds and wheezing. Chest x-ray study shows diffuse interstitial infiltrates. Laboratory studies show the following: white blood cell count 21,000/mm³; hemoglobin 9.5 g/dL; hematocrit 29%; platelet count 45,000/mm³; prothrombin time 18 sec; partial thromboplastin time 42 sec; fibrinogen 180 mg/dL; and D-dimer 1500 ng/mL. Which of the following is the most appropriate initial step? A. Administration of fresh frozen plasma B. Administration of packed red blood cells C. Administration of vitamin K D. Emergent intubation E. Endoscopy and colonoscopy

Intravenous calcium gluconate **has severe hyperkalemia

A 61-year-old woman comes to the emergency department because she has had shortness of breath for the past three days. Medical history includes end-stage kidney disease, and she says she missed her two most recent hemodialysis treatments. She is afebrile; pulse rate is 94/min, respirations are 24/min, and blood pressure is 148/96 mmHg. Oxygen saturation is 95% on 2 L oxygen. Physical examination shows crackles in the lung bases, bilaterally. Basic metabolic panel shows a potassium level of 6.9 mEq/L. Electrocardiography shows peaked T waves. The patient is admitted to the hospital, and the medical team is informed that it will be one hour before hemodialysis can be initiated in this patient. Which of the following is the most appropriate next step in management? A. Intravenous 0.9% saline B. Intravenous calcium gluconate C. Intravenous insulin D. Oral sodium polystyrene sulfonate E. Supportive care

Negatively birefringent, needle-shaped rate crystals **gouty arthritis from drinking alcohol or taking thiazide diuretics

A 62-year-old man comes to the internal medicine clinic because he had acute onset of severe pain in the right great toe that began overnight, about six hours ago. The patient says the pain is so extreme that he was unable to put his sock or shoe on his right foot. He has not had trauma to the toe, fever, chills, or previous similar episodes, but he says that he recently began taking hydrochlorothiazide for treatment of high blood pressure. On physical examination, the first metacarpophalangeal joint of the right foot is erythematous and exquisitely tender to touch. Aspiration of the joint is performed. Analysis of the aspirate is most likely to show which of the following? A. Negatively birefringent, needle-shaped urate crystals B. Positively birefringent, rhomboid-shaped calcium pyrophosphate dehydrate crystals C. White blood cell count >50,000/mm³ with >75% polymorphonuclear leukocytes D. White blood cell count >2000/mm³ but less than 50,000/mm³ E. White blood cell count <200/mm³

Cystoscopy with biopsy **bladder cancer until proven otherwise

A 62-year-old man with benign prostatic hyperplasia comes to the clinic because he had one episode of painless hematuria two days ago. He says he had a red tinge in his urine during that episode but has not noted this in his urine since that time. The patient has not had dysuria, fever, increased urinary frequency, nocturia, or recent trauma. He has smoked two packs of cigarettes daily for the past 30 years. Temperature is 36.1°C (97.0°F), pulse rate is 75/min, respirations are 16/min, and blood pressure is 133/83 mmHg. On physical examination, the prostate is moderately enlarged, but no nodules are noted. Laboratory studies show a prostate-specific antigen level of 2.8 ng/mL. Urinalysis shows the following: protein 1+, white blood cells 2/hpf, blood 3+, red blood cells too numerous to count, no casts, nitrites negative, and leukocyte esterase negative. CT scan of the abdomen is ordered. Which of the following is the most appropriate next step? A. Biopsy of the prostate B. Ciprofloxacin therapy C. Cystoscopy with biopsy D. Finasteride therapy E. Transrectal ultrasonography

Diverticulitis

A 63-year-old man comes to the emergency department because he has had worsening pain in the left lower quadrant of the abdomen over the past four days. He has had some diarrhea without blood and has felt feverish. Today, he says he developed urinary discomfort. Temperature is 38.3°C (101.0°F), pulse rate is 110/min, and blood pressure is 98/58 mmHg. Physical examination shows tenderness on palpation of the left lower abdomen with rigidity and guarding. A mass is palpated in the area of tenderness. White blood cell count is 18,000/mm³, and lactic acid is 3.1 mmol/L; hemoglobin and hematocrit are within normal limits. Urinalysis shows no abnormalities. Which of the following is the most likely diagnosis? A. Colon cancer B. Diverticulitis C. Large bowel obstruction D. Left-sided nephrolithiasis E. Urinary tract infection

Ammonia **hepatic encephalopathy

A 63-year-old man is brought to the emergency department by ambulance because he has had altered mental status for the past four days. His partner says that he seemed more forgetful over the past few days, and she thought he might not be feeling well because he seemed to be eating and drinking less. Medical history includes hypertension, heart failure, and cirrhosis. Pulse rate is 90/min, respirations are 20/min, and blood pressure is 105/72 mmHg. The patient is disoriented and slow to respond. He is alert to name but not to time, place, or situation. On physical examination, mucous membranes are dry, the skin and eyes are jaundiced, and asterixis is noted. Which of the following laboratory values is most likely to be abnormal? A. Ammonia B. Glucose C. Salicylate D. Thyroid-stimulating hormone E. White blood cells

Peptic ulcer disease

A 65-year-old man comes to the emergency department because he has had worsening fatigue over the past two days. He says his stool is dark in color and sticky in consistency. He does not have pain or discomfort. The patient recently sustained a back injury and has been taking over-the-counter nonsteroidal anti-inflammatory drugs for the past 10 days. He says he has never drunk alcohol. Temperature is 37.0°C (98.6°F), pulse rate is 115/min, and blood pressure is 130/80 mmHg. The patient appears pale but is not in acute distress. Laboratory studies show a hemoglobin level of 7.5 g/dL, hematocrit of 25%, creatinine level of 0.9 mg/dL, and urea nitrogen level of 55 mg/dL. Result of stool guaiac test is positive. Which of the following is the most likely cause? A. Colon cancer B. Crohn disease C. Diverticular bleeding D. Peptic ulcer disease E. Variceal bleeding

Ankle-brachial index **peripehral artery disease

A 65-year-old man with a history of hypertension, hyperlipidemia, and tobacco use comes to the clinic because he has had pain in the left calf during the past three months. He works as a mail carrier and says that the pain starts after he walks about a half mile and resolves if he rests for about 15 minutes, at which time he is able to resume walking. Temperature is 37.1°C (98.8°F), pulse rate is 78/min, respirations are 16/min, and blood pressure is 126/88 mmHg. Oxygen saturation is 98% on room air. On physical examination, the left lower extremity is dry, shiny, and hairless. It is slightly cooler to touch than the right. Based on these findings, which of the following is the most appropriate initial study? A. Ankle-brachial index B. Arteriography C. Complete blood cell count D. D-dimer assay E. X-ray studies of the left leg

Increased pressure in the pulmonary arteries ** pulmonary hypertension is evident from SOB with edema and JVD

A 65-year-old woman comes to the emergency department because she has had worsening shortness of breath and fatigue over the past six months. She also has had edema in her lower legs for the past two weeks and generalized edema for the past two days. She has not had fever, chills, or chest pain. Medical history does not include any lung conditions, but she has diabetes mellitus for which she takes metformin. She has never smoked cigarettes, does not drink alcoholic beverages, and does not use illicit drugs. Temperature is 36.8°C (98.2°F), pulse rate is 90/min, and blood pressure is 90/54 mmHg. Oxygen saturation is 91%. Physical examination shows generalized edema. Auscultation of the lungs shows crackles in both lung bases. Heart sounds are regular with no murmurs. Jugular venous distention is noted. Moderate hepatomegaly is noted as well as 2+ peripheral edema up to the knees. Laboratory studies show brain natriuretic peptide of 410 pg/mL and troponin of 0.4 µg/mL. Electrocardiography shows right axis deviation with upright R waves in V1 and V2 with deep S waves in V5-V6. Transthoracic echocardiography shows an ejection fraction of >60% and pulmonary artery pressure of 75 mmHg. Which of the following is the most likely explanation of these findings? A. Bacterial infection in the alveoli B. Decreased pressure in the portal veins C. Fluid overload due to heart failure with reduced ejection fraction D. Increased pressure in the pulmonary arteries E. Ischemia of myocardial tissue

Irritable bowel syndrome **associated with fibromyalgia

A 65-year-old woman comes to the internal medicine clinic because she has had widespread muscular pain for the past four months which started primarily in her neck and shoulders. She says she also has had fatigue, headaches, and poor sleep during this time. The patient is also concerned that her memory is worsening. Physical examination shows diffuse bilateral tenderness to palpation in the neck, shoulder girdle, and upper back. On laboratory studies, C-reactive protein level and erythrocyte sedimentation rate are within normal limits. Which of the following conditions is most likely associated with this patient's disorder? A. Celiac disease B. Diverticulosis C. Gallstones D. Inflammatory bowel disease E. Irritable bowel syndrome

Tyrosine kinase inhibitors **idiopathic pulmonary fibrosis should be treated with TKIs (pirfenidone or nintedanib)

A 66-year-old man comes to the clinic because he has had worsening shortness of breath over the past year that is not associated with cough, fever, hemoptysis, or illness. He has not had any swelling or chest pain. He has no history of recent travel and has never owned birds or been exposed to asbestos, mold, or metals. He has no history of rheumatoid arthritis or dry eyes or mouth. The patient says he was recently evaluated by rheumatologist, and all test results were negative. He takes no medications. He has never smoked, does not drink alcoholic beverages, and does not use illicit drugs. Temperature is 36.6°C (97.8°F), pulse rate is 90/min, respirations are 16/min, and blood pressure is 123/78 mmHg. Oxygen saturation is 88% on room air. The patient does not appear to be in acute distress. On physical examination, mucus membranes are pink and wet. Heart rate is regular and no murmurs, gallops, or rubs are noted. Auscultation of the lungs shows crackles during inspiration throughout the lung fields. No peripheral edema is noted. Chest x-ray study is nonspecific. CT scan of the chest shows an unusual interstitial pneumonia pattern with predominant basilar honeycombing. Which of the following medication classes is most likely to slow progression of this patient's disease? A. Antibiotics B. Anticoagulants C. Corticosteroids D. Immunosuppressants E. Tyrosine kinase inhibitors

Rest and non steroidal anti-inflammatory drugs **prepatellar bursitis from micro trauma

A 66-year-old man comes to the internal medicine clinic because he had sudden onset of swelling of his left knee one day ago. He works as a plumber and does not have any other symptoms. Temperature is 37.0°C (98.6°F), pulse rate is 68/min, respirations are 16/min, and blood pressure is 128/85 mmHg. Physical examination of the left knee shows a 6-cm fluid-filled mass that is tender to palpation. No erythema or warmth is noted. Which of the following is the most appropriate initial management? A. Aspiration of the fluid B. Incision and drainage C. Injection of corticosteroids D. Rest and nonsteroidal anti-inflammatory drugs E. Surgical excision

Catheter-based thrombectomy **acute limb ischemia (this is the most definitive intervention but could use unfractionated heparin initially)

A 67-year-old man is being admitted to the hospital because he had acute onset of severe pain as well as numbness in the left leg two hours ago. Medical history includes peripheral arterial disease and patent foramen ovale. The patient is afebrile; pulse rate is 102/min and irregularly irregular, respirations are 16/min, and blood pressure is 110/62 mmHg. On physical examination, the left leg appears pale with mottled skin. In the right leg, 2+ pulses are noted; no dorsalis pedis or posterior tibialis pulses are noted in the left leg. Sensation to soft or sharp touch is absent in the left leg. Which of the following definitive interventions is the most appropriate to improve the viability of this patient's left leg? A. Bypass grafting B. Catheter-based thrombectomy C. Clopidogrel D. Unfractionated heparin E. Warfarin

Voriconazole **pulmonary aspergillosis fungal ball from being immunocompromised

A 67-year-old man is being evaluated in the hospital for pneumonia and sepsis. He was brought to the emergency department because he had sudden onset of shortness of breath after having worsening shortness of breath with cough and congestion over the past six days. Laboratory studies obtained in the emergency department showed white blood cell count of 21,000/mm³ and lactic acid level of 2.9 mEq/L; procalcitonin level was within normal limits. A CT scan of his chest showed a dense, well-circumscribed lesion with a surrounding halo of ground glass attenuation with air-crescent sign with a cavity. The patient's symptoms have not responded to intravenous vancomycin and piperacillin/tazobactam. Medical history includes renal transplantation, and medications include tacrolimus and mycophenolate. Current temperature is 38.3°C (101.0°F), pulse rate is 88/min, respirations are 16/min, and blood pressure is 110/80 mmHg. Oxygen saturation is 94% on 2 L of oxygen via nasal cannula. The patient appears ill but is not in acute distress. Auscultation of the lungs shows scattered crackles without wheezes. No peripheral edema is noted. Which of the following additional treatments is most appropriate? A. Cefepime B. Methylprednisolone C. Nystatin D. Oseltamivir E. Voriconazole

Dual-energy x-ray absorptiometry of the hip and lumbar spine **DEXA scan in women over 65 and postmenopausal women < 65 for osteoporosis

A 67-year-old woman comes to the internal medicine clinic for routine physical examination. Medical history includes hypertension; she is otherwise healthy. The patient has a history of cigarette smoking. Body mass index is 19 kg/m². Temperature is 37.1°C (98.8°F), pulse rate is 80/min, respirations are 16/min, and blood pressure is 128/87 mmHg. Oxygen saturation is 98% on room air. During the interview, she says that her friend recently fell and fractured a hip, and she wonders if she has an increased risk of hip fracture. Based on this concern, it is most appropriate to recommend she undergo which of the following screening tests? A. CT scan of the hip and lumbar spine B. Dual-energy x-ray absorptiometry of the hip and lumbar spine C. MRI of the hip and lumbar spine D. Ultrasonography of the hip and lumbar spine E. X-ray studies of the hip and lumbar spine

Peaked T waves

A 67-year-old woman comes to the internal medicine clinic to discuss recent results of routine laboratory studies. Medical history includes no chronic disease conditions, and she is asymptomatic. Temperature is 37.0°C (98.6°F), pulse rate is 68/min, respirations are 16/min, and blood pressure is 126/84 mmHg. Laboratory studies show a serum potassium level of 5.6 mEq/L. Based on this finding, which of the following additional findings is most likely on electrocardiography? A. Delta wave B. Narrow QRS C. Peaked T waves D. Prolonged QT interval E. ST elevation

Prerenal azotemia **caused by decreased renal perfusion, commonly as a result of hypovolemia and has evidence of hypotension

A 68-year-old man comes to the emergency department because he has felt dizzy and light-headed for the past one week. Medical history includes hypertension, and he is compliant with hydrochlorothiazide therapy. The patient has not had any recent infections or changes to his medications. Blood pressure is 74/42 mmHg, and orthostatic blood pressures are positive. Laboratory studies show blood urea nitrogen of 32 mg/dL and creatinine of 1.5 mg/dL (baseline creatinine is 0.8 mg/dL). Urinalysis show no casts but increased specific gravity. Intravenous fluids are administered, and the patient is admitted to the internal medicine service. Subsequent laboratory studies show blood urea nitrogen of 26 mg/dL and creatinine 1.1 mg/dL. Which of the following is the most likely explanation for the acute kidney injury in this patient? A. Acute glomerulonephritis B. Acute interstitial nephritis C. Acute tubular necrosis D. Nephrotic syndrome E. Prerenal azotemia

Duplex ultrasonography of the neck **carotid stenosis

A 68-year-old woman comes to the internal medicine clinic for routine follow-up examination. She is asymptomatic, and vital signs are within normal limits. On physical examination, a carotid bruit is heard. Which of the following studies is the most appropriate next step in diagnosis? A. Cerebral angiography B. CT scan of head C. Duplex ultrasonography of the neck D. MRI of the head and neck E. PET scan of the brain

Cardiac tamponade **pulsus paradoxus

A 69-year-old woman comes to the emergency department because she has had worsening dizziness and dyspnea on exertion for the past four days. Medical history includes breast cancer, and she has a history of cigarette smoking. Pulse rate is 118/min, respirations are 26/min, and blood pressure is 82/40 mmHg. Oxygen saturation is 88% on room air. Physical examination shows distant heart sounds. Blood pressure markedly decreases with inspiration. Which of the following is the most likely diagnosis? A. Acute decompensated heart failure B. Cardiac tamponade C. Myocarditis D. Pulmonary embolism E. Restrictive cardiomyopathy

Airway narrowing and decreased lung recoil **COPD exacerbation

A 70-year-old man comes to the clinic because he has had dyspnea and cough during the past two weeks. He says the cough produces sputum that is clear but sometimes green in the morning. The patient has had difficulty performing his normal activities due to shortness of breath. He has not had any fevers or chills. Medical history does not include any lung conditions. He has smoked one pack of cigarettes per day since he was 20 years old. The patient does not drink alcohol or use illicit drugs. Temperature is 37.2°C (99.0°F), pulse rate is 115 /min, respirations are 30/min, and blood pressure is 126/66 mmHg. Oxygen saturation is 89% on room air. The patient appears thin and is leaning forward with his hands on his knees. On physical examination, decreased breath sounds are noted at the lung bases with generalized wheezing and rhonchi. No edema or skin changes are noted. Chest x-ray study shows hyperinflation with flattening of the diaphragm and no consolidation. Bedside pulmonary function testing shows a ratio of forced expiratory volume in one second to forced vital capacity (FEV1/FVC) of <70% and FVC of <80%. These measurements do not improve after administration of a bronchodilator. Which of the following is the most likely cause of the findings in this patient? A. Airway narrowing and decreased lung recoil B. Mucus hypersectretion and smooth muscle contraction C. Mutation in the CFTRgene and defective chloride conductance D. Thrombophilia and mismatched ventilation/perfusion ratio E. Viral infiltration of lung mucosa and inflammation

Papillary muscle rupture **papillary muscle rupture is a serious complication that typically occurs 2-7 days following an MI (diagnosis made by echocardiogram and definitive treatment is surgical repair)

A 70-year-old man is brought to the emergency department by ambulance because he had acute onset of shortness of breath and light-headedness one day ago. He was discharged from the hospital five days ago after he sustained an acute myocardial infarction, for which he received a coronary stent. Pulse rate is 99/min, respirations are 21/min, and blood pressure is 76/54 mmHg; he is afebrile. Oxygen saturation is 93% on room air. Physical examination shows a grade 2/6 mid systolic murmur that was not documented on the patient's most recent admission to the hospital. Auscultation of the lungs shows diffuse wet crackles, bilaterally. Current electrocardiography is unchanged from discharge from the hospital five days ago. Which of the following is the most likely etiology of the new murmur in this patient? A. Dressler syndrome B. Infectious endocarditis C. Papillary muscle rupture D. Recurrent myocardial infarction E. Rheumatic fever

Alendronate **Paget disease of the bone from abnormal bone remodeling

A 70-year-old man returns to the internal medicine clinic to discuss the results of laboratory studies that were obtained after he was evaluated for bone pain one week ago. Laboratory studies show alkaline phosphatase level of 304 U/L, calcium level of 9.2 mg/dL, and phosphorous level of 4.0 mg/dL. An x-ray study of his pelvis is shown. Based on these findings, which of the following is the most appropriate initial treatment? A. Alendronate B. Calcium C. Raloxifene D. Teriparatide E. Vitamin D

Chronic lymphocytic leukemia **skin cancer is a frequent complication of CLL

A 71-year-old man comes to the clinic for routine follow-up and healthcare maintenance. During the interview, he says he is concerned about some recent skin changes on his left ear. Medical history includes chronic lymphocytic leukemia, which is stable and monitored by an oncologist, hypertension, for which he takes lisinopril, hyperlipidemia, for which he takes a statin, multiple sclerosis, which is stable and monitored by a neurologist, and eczema, which flares occasionally and is treated with moisturizers. The patient has a history of moderate sun exposure when he was younger but tries to use sunscreen regularly now. Family history does not include skin cancer. On physical examination, crusty white lesions are noted on the left ear. Which of this patient's medical conditions increases his risk for skin cancer? A. Chronic lymphocytic leukemia B. Eczema C. Hyperlipidemia D. Hypertension E. Multiple sclerosis

Inferior wall **ST elevation in leads II, III, and aVF with reciprocal ST depression in aVL

A 72-year-old man comes to the emergency department because he had acute onset of retrosternal chest pain radiating to his left arm and jaw as well as nausea and diaphoresis one hour ago. Family history includes early coronary artery disease. Electrocardiography is shown. The patient receives urgent reperfusion and is admitted to the inpatient medicine service for close observation. This patient has most likely sustained which of the following types of myocardial infarction? A. Anterior wall B. Inferior wall C. Lateral wall D. Posterior wall E. Septal

Panarteritis affecting the media with CD+ and macrophages **temporal arteritis/giant cell arteritis is a systemic inflammatory vasculitis that causes monocular vision, jaw claudication, and systemic symptoms (corticosteroids are the mainstay of treatment)

A 72-year-old man comes to the emergency department because he has had vision loss in his right eye for the past 24 hours. He says he had development of a persistent mild headache two months ago, and he now has fatigue, fever, and decreased appetite. He also says that when he tried to eat a steak yesterday, he had a very hard time chewing because of pain in his jaw. The patient does not have paralysis, weakness, or changes in mentation. He has no history of similar symptoms in the past, and he does not smoke cigarettes, drink alcoholic beverages, or use illicit drugs. Temperature is 38.2°C (100.8°F), and pulse rate is 102/min. The patient is oriented to person, place, time, and situation. Physical examination of the right temple shows an area of mild erythema with thickening and tenderness to palpation. Cranial nerves are intact; however, complete vision loss in the right eye is noted. Romberg test is negative. No weakness or numbness is noted in the extremities. Biopsy of the temporal artery is planned. Which of the following biopsy results is most likely? A. Clusters of basiloid cells with palisading B. Keratinized squamous epithelium with fibroelastic connective tissue C. Multinucleated giant cells with positive Tzanck test D. Panarteritis affecting the media with CD+ and macrophages E. Spongiosis with intercellular edema

Prescribe atorvastatin ** TIA are at very high risk for recurrence or progression to full CVA. mainstay of treatment is to decrease the risk of further progression

A 72-year-old man is admitted to the hospital after he was brought to the emergency department by ambulance immediately after he had onset of right-sided weakness. He says he had complete loss of function in his right side that was accompanied by difficulty speaking. After two hours in the emergency department, the patient's symptoms completely resolved. He has never had similar symptoms in the past. Medical history includes no chronic disease conditions, and he takes no medications. The patient quit smoking 10 years ago, drinks alcoholic beverages occasionally, and does not use illegal drugs. Blood pressure is 110/78 mmHg. Physical examination shows no carotid bruits, and heart sounds are regular without murmurs, gallops, or rubs. No peripheral edema is noted. Neurological examination shows no focal deficits. Results of complete blood cell count and basic metabolic panel are within normal limits. Total cholesterol level is 200 mg/dL, high-density lipoprotein cholesterol level is 29 mg/dL, and low-density lipoprotein cholesterol level is 130 mg/dL. Electrocardiography shows a normal sinus rhythm at 80/min. CT scan of the head shows no intracranial hemorrhage or other abnormalities. Results of an MRI of the brain are pending. Which of the following interventions is most appropriate to avoid further complications in this patient? A. Order Doppler ultrasonography of the lower extremities B. Prescribe atorvastatin C. Prescribe metoprolol D. Prescribe warfarin E. Refer for physical and occupational therapy

Multiple Myeloma **this patient has monoclonal gammopathy of undetermined significance (MGUS)

A 72-year-old man who comes to the clinic for routine physical examination says that he has had worsening fatigue and some slight tingling in his feet over the past three months. He also has had a 10-lb unintentional weight loss during this time. The patient has not had fevers, chills, or gastrointestinal bleeding. Medical history includes no chronic disease conditions, and he takes no medications. Temperature is 37.0°C (98.6°F), pulse rate is 90/min, and blood pressure is 128/88 mmHg. Physical examination shows no abnormalities. Laboratory findings include the following: white blood cell count 8/mm³; hemoglobin 11 g/dL; hematocrit 35%; mean corpuscular volume 90 µm³; mean corpuscular hemoglobin 30 pg/cell; mean corpuscular hemoglobin concentration 33 g/dL; platelet count 250/mm³; erythrocyte sedimentation rate 20 mm/hr; albumin-globulin ratio 0.5; thyroid-stimulating hormone 3 µU/mL; and vitamin B12 250 pg/mL. Further workup with serum protein electrophoresis shows elevated M protein. Based on these findings, this patient is at increased risk of developing which of the following conditions? A. Acquired immunodeficiency syndrome B. Hepatocellular carcinoma C. Meningococcal meningitis D. Micronutrient malnutrition E. Multiple myeloma

Piperacillin/tazobactam and ciprofloxacin **community-acquired pneumonia with risk factors for pseudomonas

A 76-year-old man comes to the emergency department because he has had cough, fever, and shortness of breath for the past two weeks. He says his symptoms started as a mild cold with a stuffy nose and cough, but over the past two days, he has had worsening shortness of breath. Medical history includes diabetes mellitus, hypertension, high cholesterol, and rheumatoid arthritis. He takes metformin, atorvastatin, lisinopril, and mycophenolate. He used to smoke cigarettes but quit five years ago; he does not drink alcoholic beverages or use illicit drugs. Temperature is 39.0°C (102.2°F), pulse rate is 110/min, respirations are 20/min, and blood pressure is 110/80 mmHg. Oxygen saturation is 89% on room air and 94% on 2 L of oxygen via nasal canula. On physical examination, mucous membranes are dry and red. Auscultation of the lungs shows E to A changes and diffuse crackles on the right. Chest x-ray study is shown. Result of a nasal swab is negative for methicillin-resistant Staphylococcus aureus. Laboratory findings include the following: white blood cell count 16,000/mm³ with 95% neutrophils; immunoglobulin 2.1 g/L; creatinine 2.1 mg/dL; urea nitrogen 44 mg/dL; lactic acid 3.1 mEq/L; and procalcitonin is 0.15 ng/mL. Which of the following is the most appropriate initial treatment? A. Ampicillin/sulbactam and azithromycin B. Ceftriaxone and doxycycline C. Oseltamivir and prednisone D. Piperacillin/tazobactam and ciprofloxacin E. Piperacillin/tazobactam and vancomycin

Inhalation of asbestos

A 76-year-old man comes to the primary care clinic because he has had worsening shortness of breath over the past year. He says that he has been having a hard time walking up flights of stairs or around the grocery store because he becomes dyspneic. He has not had associated cough, wheezing, or fever, and he has no history of similar symptoms. The patient believes his symptoms are a sign that he is getting older. He has not taken anything for the symptoms but has altered his activities to avoid feeling short of breath. Medical history includes prostate cancer treated with prostatectomy, high blood pressure, and high cholesterol. Current medications include lisinopril, atorvastatin, and baby aspirin. Family medical history does not include similar symptoms. He worked as a shipyard engineer in the navy for 20 years before retiring and then working as a diesel mechanic. The patient has never smoked cigarettes, drinks alcoholic beverages occasionally, and has never used illicit drugs. Temperature is 37.0°C (98.6°F), pulse rate is 90/min, and blood pressure is 130/56 mmHg. Oxygen saturation is initially 88% but improves to 92% with rest. On physical examination, auscultation of the lungs shows decreased chest expansion and bibasilar rales most noticeable over the lateral bases. Prominent clubbing is noted. No edema or jugular venous distention is noted. Bedside pulmonary function tests show decreases in FVC, FEV1, and TLC as well as a normal FEV1/FVC ratio. Which of the following mechanisms is most likely causing the symptoms in this patient? A. Bronchospasm B. Clotting of a pulmonary artery C. Inhalation of asbestos D. Prostate cancer cell infiltration E. Streptococcus pneumoniaeinfiltration of alveoli

Systolic crescendo-decrescendo murmur heard best at the right upper sternal border

A 78-year-old man comes to the clinic to establish care. On review of the medical records from his previous provider, aortic stenosis is listed in his history. On auscultation of this patient's heart, which of the following findings is most likely? A. Blowing holosystolic murmur heard best at the apex B. Diastolic decrescendo blowing murmur heard best at the left upper sternal border C. Diastolic rumble heard best at the apex D. Harsh mid systolic ejection crescendo-decrescendo murmur heard best at the left upper sternal border E. Systolic crescendo-decrescendo murmur heard best at the right upper sternal border

Duration of the seizure **status epilepticus lasts longer then 5 minutes

A 78-year-old man is admitted to the ICU after a bystander saw him fall in the grocery store. At the scene, the patient appeared to be unconscious but had sporadic twitching. In the emergency department, the seizure subsided with administration of lorazepam. Which of the following components of this patient's history is most likely to support a diagnosis of status epilepticus? A. Duration of the seizure B. History of previous seizures C. Interval of time it took to regain normal mental status D. Presence of an aura E. Tonic-clonic features

Helicobacter pylori gastritis

A 78-year-old man with dementia who lives in a home care facility is brought to the clinic because he has abdominal pain. His primary caretaker is not present, he has not brought any paperwork with him, and he is only able to answer minimal questions. The patient says it hurts all over his abdomen, although he cannot describe the pain. He does not know how long it has been bothering him. Vital signs are within normal limits. On physical examination, there is tenderness to palpation of the epigastric area. The physician assistant contacts the home care facility and determines that the patient was evaluated in the emergency department one week ago. After tests results were obtained, therapy with clarithromycin, amoxicillin, and omeprazole was initiated three days ago. The facility staff do not know the diagnosis. Which of the following is the most likely cause of the abdominal pain in this patient? A. Bacteroides fragilisdiverticulitis B. Candida albicansesophagitis C. Escherichia colipancreatitis D. Helicobacter pylori gastritis E. Klebsiella pneumoniaeascending cholangitis

Short-acting beta-agonist as needed with scheduled budesonide/formoteral **moderate persistent asthma given daily symptoms and is waking up frequently due to SOB

An 18-year-old man comes to the clinic because he has had worsening shortness of breath over the past few years. He says his symptoms have worsened significantly over the past month. The shortness of breath woke him up two times this past week, and he has been unable to continue a normal level of activity. The symptoms are usually worse with exercise but recently have become severe even during rest. The patient has been using his friends inhaler which has provided some relief. Medical history includes no chronic disease conditions, and he takes no medications. He does not smoke cigarettes, drink alcoholic beverages, or use illicit drugs. Temperature 36.7°C (98.0°F), pulse rate is 66/min, respirations are 12/min, and blood pressure is 110/80 mmHg. Oxygen saturation is 97%. The patient does not appear to be in acute distress. On physical examination, heart rate is regular with no murmurs, gallops, or rubs. Auscultation of lungs shows scattered wheezes throughout the lung fields. Which of the following medication regimens is the most appropriate initial step in treatment? A. Short-acting beta-agonist as needed with a scheduled long-acting beta-agonist B. Short-acting beta-agonist as needed with scheduled budesonide/formoterol C. Short-acting beta-agonist as needed with tiotropium D. Short-acting beta-agonist every two hours E. Short-acting beta-agonist scheduled with long-term oral prednisone daily

Diphtheria

An 18-year-old woman comes into the clinic because she has had a sore throat, fevers, and chills for the past four days. She says the symptoms started four days after she returned from a trip to Indonesia. She grew up in a family that did not believe in vaccinations, and she says she rarely saw a medical provider as a child. The patient has had sexual intercourse with two new partners during the past four weeks. Temperature is 38.1°C (100.6°F). Physical examination shows a thick, gray, adherent pseudomembrane on the tonsils and throat. Diffuse cervical lymphadenopathy is also noted. Gram stain of a throat culture shows nonencapsulated, nonmotile, club-shaped gram-positive bacilli. Which of the following is the most likely diagnosis? A. Chlamydia B. Diphtheria C. Influenza D. Mumps E. Streptococcal pharyngitis

Wolff-Parkinson-White syndrome ** type of SVT characterized by delta wave, slurred QRS upstroke, and short PR interval and wide QRS

An otherwise healthy 27-year-old patient who identifies as gender nonconforming comes to the clinic because of intermittent lightheadedness and palpitations during the past three weeks. Electrocardiography is shown. Which of the following is the most likely diagnosis? A. Atrial fibrillation B. Brugada syndrome C. Long QT syndrome D. Premature ventricular complexes E. Wolff-Parkinson-White syndrome


Related study sets

IST-294 Ethical Hacking-Chapter 4 Footprinting and Social Engineering

View Set

ACG2071 - Chapter 9: Flexible Budgets, Standard Costs and Variances

View Set

Texas Promulgated Contract Forms Chapter 7

View Set

Accounting Chapter Two True/False Questions

View Set

Chp 4 fluid and electrolyte - feel free to add if you are in NURS 125 at MC

View Set

Инструкция по организации и обслуживанию воздушного движения

View Set